Finding an explicit formula for the sequence of partial sums

Click For Summary
To find an explicit formula for the sequence of partial sums of the series sum from n=1 to infinity of 1/(n(n+1)), it can be reformulated as a telescoping series: sum from n=1 to infinity of (1/n - 1/(n+1)). The first few terms demonstrate that all terms cancel except for 1 and 1/(n+1), leading to the partial sum formula of (1 - 1/(n+1)). The convergence of the series is confirmed by taking the limit of the partial sums, which approaches 1 as n approaches infinity. Thus, the series converges to 1.
RJLiberator
Gold Member
Messages
1,094
Reaction score
63

Homework Statement


I am trying to wrap my head around what it means to find an explicit formula for the sequence of partial sums.

Question: Find an explicit formula for the sequence of partial sums and determine if the series converges.

a) sum from n=1 to n=infinity of 1/(n(n+1))

Homework Equations

The Attempt at a Solution



This is a telescoping series.
We can reformat the sum as follows:

a) sum from n=1 to n=infinity of (1/n - 1/(n+1))

Writing out the first few terms we see
(1-1/2+1/2-1/3+1/3-1/4+1/4-...)

Clearly, everything cancels out except 1 and 1/(n+1).

The question asks for an explicit formula for the sequence of partial sums. Would that simply be (1-1/(n+1)) ?

Additionally, how can I tell that this converges (my hunch is that it does, since the terms are telescoping and canceling each other out)
 
Physics news on Phys.org
RJLiberator said:
The question asks for an explicit formula for the sequence of partial sums. Would that simply be (1-1/(n+1)) ?

Yes.

If the first index is 1 (as it is in you problem) , the "n-th partial sum" of a series ##\{a_n\}## is defined to be ##\sum_{i=1}^n a_n##. For example, the 2nd-partial sum of the series in your example is (1- 1/2) + (1/2 - 1/3), so you can check your formula agrees with that number when n = 2.

Additionally, how can I tell that this converges (my hunch is that it does, since the terms are telescoping and canceling each other out)

What do you mean by "this"? Your problem involves 3 different things that could converge or diverge.
1) The sequence
2) The sequence of partial sums of the above sequence
3) The infinite series defined by summing the sequence in 1) above.

The limit of a sequence ##\{a_n\}##whose terms are given by a formula ##a_n = f(n)## is ##lim_{n \rightarrow \infty} f(n)##.

The limit of a series ##\sum_{i=1}^\infty a_n## is given by the limit of its n-th partial sum as n approaches infinity. (Some textbooks use that statement as the definition for the limit of an infinite series.)
 
  • Like
Likes RJLiberator
Ah, the partial sum formula just clicked for me with that confirmation. I can see that works out.

As for convergence, I am learning to take the limit of the partial sum, this would converge to 1.
 
Question: A clock's minute hand has length 4 and its hour hand has length 3. What is the distance between the tips at the moment when it is increasing most rapidly?(Putnam Exam Question) Answer: Making assumption that both the hands moves at constant angular velocities, the answer is ## \sqrt{7} .## But don't you think this assumption is somewhat doubtful and wrong?

Similar threads

  • · Replies 1 ·
Replies
1
Views
2K
  • · Replies 4 ·
Replies
4
Views
2K
  • · Replies 17 ·
Replies
17
Views
2K
  • · Replies 7 ·
Replies
7
Views
2K
  • · Replies 2 ·
Replies
2
Views
2K
  • · Replies 1 ·
Replies
1
Views
2K
  • · Replies 1 ·
Replies
1
Views
1K
  • · Replies 7 ·
Replies
7
Views
1K
  • · Replies 4 ·
Replies
4
Views
2K
  • · Replies 12 ·
Replies
12
Views
3K